The list price on slacks is $22, and the list price on jumpers is $37. If Petit’s Clothing Store orders 30 pairs of slacks and 40 jumpers at a discount rate of 11%, what is the trade discount on the purchase?

Answers

Answer 1

Answer:

The trade discount is

$235.4

Step-by-step explanation:

Given data

Slacks= $22

Jumpers= $37

Number of orders for Slacks= 30

Number of orders for Jumpers= 40

Total cost

22*30=$660

37*40=$1480

=$2140

11% discount can be computed as

=11/100*2140

=$235.4


Related Questions

2(x +1) + 3(x -1) = 8(x -5)..plz solve this​

Answers

x=17/2

don't forget to follow me

Identify the outliers of the data set. Then determine if the outlier increases or decreases the value of the mean.
35, 42, 76, 38, 41, 32, 38, 36, 34, 42, 37

A: 76; decreases


B: 32; decreases


C: 32; increases


D: 76; increases

Answers

Answer:  D) 76; increases

========================================================

Explanation:

The main group or cluster of values spans from 32 to 42 (inclusive).

Then off on its own is the value 76, which we consider an outlier. This value is fairly far from the group. As a rule, large outliers pull on the arithmetic mean to make it larger than it should be. Think of it like the outlier pulling on the mean as if it was done through a magnet or gravitational pull.

Similarly, small outliers pull the mean to the left to make it smaller than it should be. We don't have any small outliers in this case.

--------------

Let's consider the set

A = {32, 34, 35, 36, 37, 38, 38, 41, 42, 42, 46}

where I've sorted the values and I replaced 76 with 46.

Computing the mean of set A gets us

(32+34+35+36+37+38+38+41+42+42+46)/11 = 38.27 approximately

--------------

Now let's form this set

B = {32, 34, 35, 36, 37, 38, 38, 41, 42, 42, 76}

which is the original set your teacher gave you. It's nearly identical to set A, except that the 46 is now 76 again.

Compute the mean of set B

(32+34+35+36+37+38+38+41+42+42+76)/11 = 41

---------------

Set A has a mean of roughly 38.72 and set B has a mean of 41. We see that the mean has increased.

The hardware store where you work charge 80 cents including tax, for a pound of nails. A customer purchases 5 3/4 pound of nails. How much should you charge the customer for nails?​

Answers

Answer:

$4.60

Step-by-step explanation:

1 lb ---> 80 cents

5 3/4 lb ---> 5 3/4 * 80 cents

[tex] 5 \dfrac{3}{4} \times 80 = [/tex]

[tex] = \dfrac{23}{4} \times \dfrac{80}{1} [/tex]

[tex] = \dfrac{23 \times 80}{4 \times 1} [/tex]

[tex] = \dfrac{1840}{4} [/tex]

[tex] = 460 [/tex]

460 cents = $4.60

5.75 x .80 = $4.60 total charge

WILL MARK YOU JF YOU HELP PLEASE HELP ME!!

Answers

Answer : D) Perpendicular to a line through a point.

You can tell this is a construction for perpendicular to a line through a point because of the arcs created around one specific point, in this case it is point P.
The answer is D hope this helped !!

Pls help pls help pls help

Answers

Answer:

tubol mo mabaho kaya 1 too 5 so it means 5 days ka sa cr nag tutubol, ok i hope it helps its the grett answer do it in your solution paper or computers

Given:
p: 2x = 16
q: 3x – 4 = 20
RE
Which is the converse of p - q?
ООО
If 2x + 16, then 3x - 47 20.
If 3x - 420, then 2x + 16.
If 2x = 16, then 3x – 4 = 20.
If 3x - 4 = 20, then 2x = 16

Answers

Given:

The given statements are:

[tex]p:2x=16[/tex]

[tex]q:3x-4=20[/tex]

To find:

The converse of [tex]p\to q[/tex].

Solution:

The statement [tex]p\to q[/tex] means if p, then q and the converse of this statement is [tex]q\to p[/tex].

[tex]q\to p[/tex] means if q , then p.

We have, [tex]p:2x=16[/tex] and [tex]q:3x-4=20[/tex].

So, the converse of given statement is:

[tex]q\to p:[/tex] If [tex]3x-4=20[/tex], then [tex]2x=16[/tex].

Therefore, the correct option is D.

Answer: Therefore, the correct option is D.

Step-by-step explanation:

Given:

p: 2x = 16

q: 3x – 4 = 20

RE

Which is the converse of p - q?

ООО

If 2x + 16, then 3x - 47 20.

If 3x - 420, then 2x + 16.

If 2x = 16, then 3x – 4 = 20.

If 3x - 4 = 20, then 2x = 16

To find:

The converse of .

Solution:

The statement  means if p, then q and the converse of this statement is .

means if q , then p.

We have,  and .

So, the converse of given statement is:

If , then .

Therefore, the correct option is D.

Divide 259875 by the smallest number so that the quotient is a perfect cube. Also find the cube root of the quotient

Answers

Answer: The smallest number by which 259875 should be divided to make it a perfect cube is 77.

Step-by-step explanation:

Let's expand the number 259875 into prime factors:

259875 = 3³ ∙ 5³ ∙ 7 ∙ 11

Since in the factorization, 7 and 11 appears only one time, we must divide the number 259875 by 7 · 11 = 77, then the quotient is a perfect cube.

259875 ÷ 77 = 3375

[tex]\sqrt[3]{3375} =\sqrt[3]{3^{3} \cdot 5^{3} } =3 \cdot 5 = 15[/tex]

2^5 + 10=?????????????

Answers

Answer:

Answer is 42

Step-by-step explanation:

2^5 + 10

2*2*2*2*2 + 10

32 + 10

42

2^5 + 10 = 42

OR

2 x 2 x 2 x 2 x 2 + 10 = 42


Remember to always multiply/divide in equation first! And to simplify the exponent.

Which graph has a slope of 4/5

Answers

Answer: whichever graph that goes up 4 and over 5

Step-by-step explanation:

rise over run, 4 up, 5 right

to find it just look at two points on the line and count how many up and how many horizontal units there are between them :)

30% of a number is x. What is 100% of the number? Assume x>0.
100% of the number is

Answers

Answer:

10x/3

Step-by-step explanation:

30% is to x as 100% is to y

We are looking for y in terms of x.

30/x = 100/y

30y = 100x

y = 100x/30

y = 10x/3

use the rules of exponents to evaluate or simplify.write without negative exponents. 1/4^-2=?

Answers

9514 1404 393

Answer:

  4² = 16

Step-by-step explanation:

The applicable rule of exponents is ...

  1/a^-b = a^b

So, ...

 [tex]\dfrac{1}{4^{-2}}=\boxed{4^2 = 16}[/tex]

_____

Additional comment

If you were to evaluate this using the Order of Operations, you would evaluate the exponent first:

  1/4^-2 = 1/(1/16)

Then, you would do the division.

 1/(1/16) = 16

__

We sometimes find it convenient to manipulate exponential terms to the form with the smallest positive exponents before we begin the evaluation.

So are you good at maths then what is
[tex]4 \times 6 + 9 - 46 + 54 - 13[/tex]
1. 71
2. 42
3. 63
4. 28
5. 35
6. 14​

Answers

maybe 28 is the answer...

Answer: 28

Order of operations is key here. So multiply first. Then you just add/subtract left to right.

24 + 9 - 46 + 54 - 13
33 - 46 + 54 - 13
-13 + 54 - 13
41 - 13
= 28

A credit card company uses these rules to calculate the minimum amount owed: For a bill of less than $100, the entire amount is due. For a bill of at least $100 but less than $500, the minimum due is $100. For a bill of at least $500 but less than $1,000, the minimum due is $300. For a bill of $1,000 or more, the minimum due is $500. Which graph shows the minimum amount due for a credit amount of x (given that the credit limit is $2,000).

Answers

1000 is the right answer

Find the sum of: 7a² - 9a + 5 and 11a + a² + 8​

Answers

Answer:

Add them:

Squares to squares, a to and number to number:

8a^2+3a+13

The simplified sum of the given expressions 7a² - 9a + 5 and 11a + a² + 8​ is 8a² + 2a + 13.

To find the sum of the expressions 7a² - 9a + 5 and 11a + a² + 8, we simply combine like terms.

The given expressions contain terms with different powers of "a."

Combining the terms with the same powers, we have:

7a² + a² = 8a² (the coefficient for the "a²" term is 7 + 1 = 8)

-9a + 11a = 2a (the coefficient for the "a" term is -9 + 11 = 2)

Finally, we combine the constant terms:

5 + 8 = 13

Putting it all together, the sum of the expressions 7a² - 9a + 5 and 11a + a² + 8 is:

8a² + 2a + 13

To learn more about polynomials click on,

https://brainly.com/question/12857688

#SPJ2


Am I correct if not plz fix it ASAP I have 4 minutes left

Answers

Answer:

no not correct it should be horizontally left 2 units and vertically down 5 units

A right triangle has side lengths 8, 15, and 17 as shown below. Use these lengths to find tanĄ, sind, and cos 4. (GIVING POINTS AND BRAINLEST TO BEST ANSWER)​

Answers

Answer:

Tan A = 8/15

Sin A = 8/17

Cos A = 15/17

I start feeling tired now

Can someone please help me?

Answers

Over 3 hours, the rain fell .75 inches. The answer is A.

Factor completely, then place the factors in the proper location on the grid. x2 - 8x + 16

Answers

Answer:

( − 4 ) 2

Step-by-step explanation:

2 − 8 + 1 6

2 -4 − 4 + 1 6

x(x-4)-4(x-4)

(x-4)(x-4)

(x-4)2

which can be also written as x-4=0

                                                   x=4

HELP ASAP WILL GIVE BRAINLIST

Consider the sequence {5,10,15,20,…}. Find n if an = 4875. Show all steps including the formulas used to calculate your answer.

Answers

Answer:

n = 975

Step-by-step explanation:

[tex]a_1 = 5 = 5 \times 1\\a_2 = 10 = 5 \times 2\\\\Therefore, \ a_n = 5 \times n\\[/tex]

[tex]Given \ a_n = 4875\\\\So, a_n = 5 \times n \\\\=> 4875 = 5 \times n\\\\=>\frac{4875}{5} = n\\\\=> 975 = n[/tex]

PLEASE ANSWER MAKE SURE YOU ARE RIGHT PLEASE I WILL MARK AS BRAINIEST

FIND THE VOLUME OF THE SPHERE

Answers

Answer:

Step-by-step explanation:

r = 1/2 unit

[tex]Volume= \frac{4}{3}\pi r^{3}\\\\=\frac{4}{3}\pi *\frac{1}{2}*\frac{1}{2}*\frac{1}{2}\\\\=\frac{1}{3}*\pi *\frac{1}{2}\\\\=\frac{1}{6}\pi[/tex]

Prove that sinxtanx=1/cosx - cosx
[tex] \sin(x) \tan(x) = \frac{1}{ \cos(x) } - \cos(x) [/tex]

Answers

Answer:

See below

Step-by-step explanation:

We want to prove that

[tex]\sin(x)\tan(x) = \dfrac{1}{\cos(x)} - \cos(x), \forall x \in\mathbb{R}[/tex]

Taking the RHS, note

[tex]\dfrac{1}{\cos(x)} - \cos(x) = \dfrac{1}{\cos(x)} - \dfrac{\cos(x) \cos(x)}{\cos(x)} = \dfrac{1-\cos^2(x)}{\cos(x)}[/tex]

Remember that

[tex]\sin^2(x) + \cos^2(x) =1 \implies 1- \cos^2(x) =\sin^2(x)[/tex]

Therefore,

[tex]\dfrac{1-\cos^2(x)}{\cos(x)} = \dfrac{\sin^2(x)}{\cos(x)} = \dfrac{\sin(x)\sin(x)}{\cos(x)}[/tex]

Once

[tex]\dfrac{\sin(x)}{\cos(x)} = \tan(x)[/tex]

Then,

[tex]\dfrac{\sin(x)\sin(x)}{\cos(x)} = \sin(x)\tan(x)[/tex]

Hence, it is proved

sketch a system of linear equation whose solution is (3,6)​

Answers

Answer: x+y = 9, 2x+3y = 24

Step-by-step explanation:

In a geometric sequence, the term an+1 can be smaller than the term ar O A. True O B. False​

Answers

9514 1404 393

Answer:

  True

Step-by-step explanation:

In a geometric sequence, the terms a[n+1] and a[n] are related by the common ratio. If the sequence is otherwise unspecified, two sequential terms may have any a relation you like.* Either could be larger or smaller than the other.

__

* If one is zero, the other must be as well. Multiplying 0 by any finite common ratio will give zero as the next term.

Find the area of the circle. Leave your answer in terms of T.

Answers

Answer:

4.2025 [tex]\pi m^{2}[/tex]

Step-by-step explanation:

Find the interest earned on $1,000 for 1 year at a 6% rate of interest when the interest is compounded quarterly.

Answers

Answer:

1060

Step-by-step explanation:

What is 34% plus 29% times 60

Answers

Answer:

34%+29%=63% 63%x60=37.8

Step-by-step explanation:

because math

Answer:

[tex] { \tt({34\% + 29\%) \times 60}} \\ = 63\% \times 60 \\ = \frac{63}{100} \times 60 \\ = 37.8[/tex]

2 divided by 0.75 full divison work i dont just need the answer​

Answers

Answer:

0.375

Step-by-step explanation:

Check the picture below.

whenever we do division of decimals, we have to mind how many decimals are there on each amount, the dividend as well as the divisor, that way we pad with zeros the other amount accordingly whilst losing the dot, for example, to say divide 3 by 0.123, 3 has no decimals, whilst 0.123 has three decimals, so we can just divide 3000 by 0123, so dividing 3 by 0.123 is the same as dividing 3000 by 123.  Another example, if we were to divide say 23.761 by 555.89331, the dividend has 3 decimals, that means 3 zeros the other way, the divisor has 5 decimals, that means 5 zeros the other way while losing the dots, so we'd end up dividing 2376100000 by 55589331000, which we can simplify to just 2376100 by 5589331, as you can see in the picture in this case.

According to the tables used by insurance companies, a 48-year old man has a 0.169% chance of
passing away during the coming year. An insurance company charges $217 for a life insurance policy
that pays a $100,000 death benefit.
What is the expected value for the person buying the insurance?

Answers

Answer:

The expected value for the person buying the insurance is of -$48.

Step-by-step explanation:

Expected value:

0.169% = 0.00169 probability of earning the death benefit of $100,000, subtracting 217, 100000 - 217 = $99,783.

100 - 0.169 = 99.831% = 0.99831 probability of losing $217.

What is the expected value for the person buying the insurance?

[tex]E = 0.00169*99783 - 0.99831*217 = -48[/tex]

The expected value for the person buying the insurance is of -$48.

17.
What is the value of the expression
2a + 5b + 3c for a = 12, b = 6, and
C=3?
A 10
B 21
C49
D 63

Answers

D. 63

2a+5b+3c

2(12)+5(6)+3(3)

24+30+9=63

Hope this helps! :)

Suppose X has an exponential distribution with mean equal to 23. Determine the following:
(a) P(X >10)
(b) P(X >20)
(c) P(X <30)
(d) Find the value of x such that P(X

Answers

Answer:

a) P(X > 10) = 0.6473

b) P(X > 20) = 0.4190

c) P(X < 30) = 0.7288

d) x = 68.87

Step-by-step explanation:

Exponential distribution:

The exponential probability distribution, with mean m, is described by the following equation:

[tex]f(x) = \mu e^{-\mu x}[/tex]

In which [tex]\mu = \frac{1}{m}[/tex] is the decay parameter.

The probability that x is lower or equal to a is given by:

[tex]P(X \leq x) = \int\limits^a_0 {f(x)} \, dx[/tex]

Which has the following solution:

[tex]P(X \leq x) = 1 - e^{-\mu x}[/tex]

The probability of finding a value higher than x is:

[tex]P(X > x) = 1 - P(X \leq x) = 1 - (1 - e^{-\mu x}) = e^{-\mu x}[/tex]

Mean equal to 23.

This means that [tex]m = 23, \mu = \frac{1}{23} = 0.0435[/tex]

(a) P(X >10)

[tex]P(X > 10) = e^{-0.0435*10} = 0.6473[/tex]

So

P(X > 10) = 0.6473

(b) P(X >20)

[tex]P(X > 20) = e^{-0.0435*20} = 0.4190[/tex]

So

P(X > 20) = 0.4190

(c) P(X <30)

[tex]P(X \leq 30) = 1 - e^{-0.0435*30} = 0.7288[/tex]

So

P(X < 30) = 0.7288

(d) Find the value of x such that P(X > x) = 0.05

So

[tex]P(X > x) = e^{-\mu x}[/tex]

[tex]0.05 = e^{-0.0435x}[/tex]

[tex]\ln{e^{-0.0435x}} = \ln{0.05}[/tex]

[tex]-0.0435x = \ln{0.05}[/tex]

[tex]x = -\frac{\ln{0.05}}{0.0435}[/tex]

[tex]x = 68.87[/tex]

Other Questions
how can I start off my informative essay of getting and staying healthy Question 1 of 10The number 3 goes on forever with no repeating pattern; therefore, it isrational.A. TrueB. FalseSUBMIT A molecule has four bonded atoms around a central atom. The central atom does not have any lone pairs of electrons. What is the geometry of the molecule? okay so, hey my name is natalie. i don't have my phone and id love to communicate with any of yall, feel free to take some points and introduce your self. :) What happens to students rights when they step onto the school grounds (list 3 things and describe it)? help me with this please Explain what the credit terms of 2/10,n/30 mean. (Check all that apply.) Multiple select question. The full payment is due within a 30-day credit period. The buyer can deduct 2% of the invoice amount if payment is made within 10 days of the invoice date. The buyer can take a discount of 10% if the invoice is paid with 30 days of the invoice date. The full payment is due within 10 days. 2. EXPLIQUE o que foi o tratado de Tordesilhas Can someone explain to me how this is wrong at all? I did the math like 10 times and somehow its wrong...? please lmk what the actual answer is because apparently im just incompetent Also giving Brainliest This is the last question from my hw Please help Ill give brainllist Find the length of side x A reaction requires 1.5 mL of ammonia if it occurs at 1.65 atm and 23 degrees celsius. If the temperature is changed to 30 degrees celsius, what will the new pressure be if the volume remains the same? when did the Jurassic Period end? A) 66 million years ago B) 144 million years ago C) 163 million years ago D) 190 million years ago Consider a species of bird that can be split into three age groupings: those aged 0-1 years, those aged 1-2 years, and those aged 2-3 years. The population is observed once a year. Given that the Leslie matrix is equal to Calculate the amount of heat needed to melt 89.9 g of solid methanol (CH3OH) and bring it to a temperature of 49.1C. Of the marbles in a bag, 4 are yellow , 4 are white , and 5 are green . Sandra will randomly choose one marble from the bag. Fill in the blanks in parts a and b. The probability that Sandra will choose a marble from the bag is subtract a2-b2 from a2 +b2 What is the energy of a photon of red light with a wavelength of 6.47 x 10-7 m ? A uniform plane electromagnetic wave propagates in a lossless dielectric medium of infinite extent. The electric field in the wave has the instantaneous expressionE(r,t) = (ix 3 - iz) 2 sin(2.10^8t + 2x/3 + 2nz/3 + 30 ), V/m. Find: a. iE, the unit vector in the direction of the wave electric fieldb. the amplitude Eo of the wavec. the wavelength of the wave d. ik, the unit vector in the direction of propagation